Difference between revisions of "2014 AMC 12B Problems/Problem 10"

m
 
(4 intermediate revisions by the same user not shown)
Line 13: Line 13:
 
Let the number of hours Danica drove be <math>k</math>. Then we know that <math>100a + 10b + c + 55k</math> = <math>100c + 10b + a</math>. Simplifying, we have <math>99c - 99a = 55k</math>, or <math>9c - 9a = 5k</math>. Thus, k is divisible by <math>9</math>. Because <math>55 * 18 = 990</math>, <math>k</math> must be <math>9</math>, and therefore <math>c - a = 5</math>. Because <math>a + b + c \leq{7}</math> and <math>a \geq{1}</math>, <math>a = 1</math>, <math>c = 6</math> and <math>b = 0</math>, and our answer is <math>a^2 + b^2 + c^2 = 6^2 + 0^2 + 1^2 = 37</math>, or <math>\boxed{D}</math>.
 
Let the number of hours Danica drove be <math>k</math>. Then we know that <math>100a + 10b + c + 55k</math> = <math>100c + 10b + a</math>. Simplifying, we have <math>99c - 99a = 55k</math>, or <math>9c - 9a = 5k</math>. Thus, k is divisible by <math>9</math>. Because <math>55 * 18 = 990</math>, <math>k</math> must be <math>9</math>, and therefore <math>c - a = 5</math>. Because <math>a + b + c \leq{7}</math> and <math>a \geq{1}</math>, <math>a = 1</math>, <math>c = 6</math> and <math>b = 0</math>, and our answer is <math>a^2 + b^2 + c^2 = 6^2 + 0^2 + 1^2 = 37</math>, or <math>\boxed{D}</math>.
  
==Solution 3 (Testing Values)==
 
We know that since adding a multiple of <math>55</math> will increase the number, we know that <math>a \leq{c}</math>. After testing values for <math>a</math>, <math>b</math>, and <math>c</math>, adding their squares, we see that using non-zero integers as the digits can yield a maximum value of <math>a^2 + b^2 + c^2</math> as 27, specifically the combination <math>(a, b, c)</math> of <math>(1, 1, 5)</math>. However, we see that no multiple of <math>55</math> can be added to <math>115</math> to achieve <math>511</math>. No combination of non-zero integers can create a higher answer, so we must try testing with a zero. Realizing that only <math>b</math> can be zero, we first try to see if <math>(1, 0, 5)</math> (which creates 26 from the expression <math>1^2 + 0^2 + 5^2</math>, answer choice A) can be added to a multiple of <math>55</math> to create <math>501</math>, which we find it cannot. However, we quickly realize that the next consecutive pair <math>(1, 0, 6)</math> does satisfy this condition, and <math>1^2 + 0^2 + 6^2 = \boxed{\textbf{(D)}\ 37}</math>. We come to this quickly after realizing that if the multiple of <math>55</math> was an even multiple, it would have to end in <math>0</math> and thus the <math>c</math> digit would remain unchanged, so it must be an odd multiple, which will carry over the <math>c</math> digit, so <math>c \geq{6}</math>.
 
 
-Solution by Joeya
 
  
 
== See also ==
 
== See also ==
 
{{AMC12 box|year=2014|ab=B|num-b=9|num-a=11}}
 
{{AMC12 box|year=2014|ab=B|num-b=9|num-a=11}}
 
{{MAA Notice}}
 
{{MAA Notice}}

Latest revision as of 19:46, 17 January 2021

Problem

Danica drove her new car on a trip for a whole number of hours, averaging 55 miles per hour. At the beginning of the trip, $abc$ miles was displayed on the odometer, where $abc$ is a 3-digit number with $a \geq{1}$ and $a+b+c \leq{7}$. At the end of the trip, the odometer showed $cba$ miles. What is $a^2+b^2+c^2?$.

$\textbf{(A)}\ 26\qquad\textbf{(B)}\ 27\qquad\textbf{(C)}\ 36\qquad\textbf{(D)}\ 37\qquad\textbf{(E)}\ 41$

Solution 1

We know that the number of miles she drove is divisible by $5$, so $a$ and $c$ must either be the equal or differ by $5$. We can quickly conclude that the former is impossible, so $a$ and $c$ must be $5$ apart. Because we know that $c > a$ and $a + c \le 7$ and $a \ge 1$, we find that the only possible values for $a$ and $c$ are $1$ and $6$, respectively. Because $a + b + c \le 7$, $b = 0$. Therefore, we have \[a^2 + b^2 + c^2 = 36 + 0 + 1 = \boxed{\textbf{(D)}\ 37}\]

Solution 2

Let the number of hours Danica drove be $k$. Then we know that $100a + 10b + c + 55k$ = $100c + 10b + a$. Simplifying, we have $99c - 99a = 55k$, or $9c - 9a = 5k$. Thus, k is divisible by $9$. Because $55 * 18 = 990$, $k$ must be $9$, and therefore $c - a = 5$. Because $a + b + c \leq{7}$ and $a \geq{1}$, $a = 1$, $c = 6$ and $b = 0$, and our answer is $a^2 + b^2 + c^2 = 6^2 + 0^2 + 1^2 = 37$, or $\boxed{D}$.


See also

2014 AMC 12B (ProblemsAnswer KeyResources)
Preceded by
Problem 9
Followed by
Problem 11
1 2 3 4 5 6 7 8 9 10 11 12 13 14 15 16 17 18 19 20 21 22 23 24 25
All AMC 12 Problems and Solutions

The problems on this page are copyrighted by the Mathematical Association of America's American Mathematics Competitions. AMC logo.png